Psych Final

¡Supera tus tareas y exámenes ahora con Quizwiz!

Irresponsible, guiltless behavior is to a client diagnosed with cluster B personality disorder as avoidant, dependent behavior is to a client diagnosed with a: 1. Cluster A personality disorder. 2. Cluster B personality disorder. 3. Cluster C personality disorder. 4. Cluster D personality disorder.

3. Cluster C personality disorder.

Consider these three drugs: divalproex (Depakote), carbamazepine (Tegretol), and gabapentin (Neurontin). Which drug also belongs to this group? 1. Clonazepam (Klonopin) 2. Risperidone (Risperdal) 3. Lamotrigine (Lamictal) 4. Aripiprazole (Abilify)

3. Lamotrigine (Lamictal)

A nurse is conducting a class for a group of newly licensed nurses on caring for clients who are at risk for suicide. Which of the following information should the nurse include in the teaching? A. A client's verbal threat of suicide is attention‐seeking behavior. B. Interventions are ineffective for clients who really want to commit suicide. C. Using the term suicide increases the client's risk for a suicide attempt. D. A no‐suicide contract decreases the client's risk for suicide.

A no‐suicide contract decreases the client's risk for suicide

A victim of a violent rape has been in the emergency department for 3hours. Evidence collection is complete. As discharge counseling begins, the victim says softly, "I will never be the same again. I can't face my friends. There is no sense of trying to go on." Select the nurse's most important response A. "Are you thinking of suicide?" B. "It will take time, but you will feel the same as before." C. "Your friends will understand when you tell them." D. "You will be able to find meaning in this experience as time goes on."

A. "Are you thinking of suicide?"

A patient says, "I've done a lot of manipulating in my relationships." Select a nonjudgmental response by the nurse. A. "How do you feel about that?" B. "It's good that you realize this." C. "That's not a good way to behave." D. "Have you outgrown that type of behavior?"

A. "How do you feel about that?"

A nurse is discussing normal grief with a client who recently lost a child. Which of the following statements made by the client indicates understanding? (Select all that apply.) A. "I may experience feelings of resentment." B. "I will probably withdraw from others." C. "I can expect to experience changes in sleep." D. "It is possible that I will experience suicidal thoughts." E. "It is expected that I will have a loss of self‐esteem."

A. "I may experience feelings of resentment." B. "I will probably withdraw from others." C. "I can expect to experience changes in sleep."

A charge nurse is discussing TMS with a newly licensed nurse. Which of the following statements by the newly licensed nurse indicates an understanding of the teaching? A. "I will schedule the client for TMS treatments 3-5 times a week for the first several weeks" B. "TMS is indicated for the clients who have schizophrenia spectrum disorders" C. "TMS treatments usually last 5-10 minutes" D. "I will provide PACU care following TMS"

A. "I will schedule the client for TMS treatments 3-5 times a week for the first several weeks"

A nurse is caring for a client who has avoidant personality disorder. Which of the following statements is expected from a client who has this type of personality disorder? A. "I'm scared that you're going to leave me." B. "I'll go to group therapy if you'll let me smoke." C. "I need to feel that everyone admires me." D. "I sometimes feel better if I cut myself."

A. "I'm scared that you're going to leave me."

Which of the following is an example of all-or-nothing thinking, which is a cognitive distortion of patients with an ED? A. "If I allow myself to gain weight, I'll be huge." B. "I'm unpopular because I'm fat." C. "When I'm thin, I'm powerful." D. "When people say I look better, they're really thinking I look fat."

A. "If I allow myself to gain weight, I'll be huge."

A nurse is caring for an adolescent client who has anorexia nervosa with recent rapid weight loss and a current weight of 90 lb. Which of the following statements indicates the client is experiencing the cognitive distortion of catastrophizing? A. "Life isn't worth living if I gain weight." B. "Don't pretend like you don't know how fat I am." C. "If I could be skinny, I know I'd be popular." D. "When I look in the mirror, I see myself as obese."

A. "Life isn't worth living if I gain weight."

A nurse is assessing a client who has major depressive disorder. The nurse should identify which of the following client statements as an overt comment about suicide? (Select all that apply.) A. "My family will be better off if I'm dead." B. "The stress in my life is too much to handle." C. "I wish my life was over." D. "I don't feel like I can ever be happy again." E. "If I kill myself then my problems will go away."

A. "My family will be better off if I'm dead." C. "I wish my life was over." E. "If I kill myself then my problems will go away."

A nurse is obtaining a nursing history from a client who has a new diagnosis of anorexia nervosa. Which of the following questions should the nurse include in the assessment? (Select all that apply.) A. "What is your relationship like with your family?" B. "Why do you want to lose weight?" C. "Would you describe your current eating habits?" D. "At what weight do you believe you will look better?" E. "Can you discuss your feelings about your appearance?"

A. "What is your relationship like with your family?" C. "Would you describe your current eating habits?" E. "Can you discuss your feelings about your appearance?"

A nurse is caring for a client who has substance‐induced psychotic disorder and is experiencing auditory hallucinations. The client states, "The voices won't leave me alone!" Which of the following statements should the nurse make? (Select all that apply.) A. "When did you start hearing these things?" B. "The voices are not real, or else we would both hear them." C. "It must be scary to hear voices." D. "Are the voices you hear telling you to hurt yourself?" E. "Why are the voices talking to only you?"

A. "When did you start hearing these things?" C. "It must be scary to hear voices." D. "Are the voices you hear telling you to hurt yourself?"

Which statement shows a nurse has empathy for a patient who made a suicide attempt? A. "You must have been very upset when you tried to hurt yourself" B. "It makes me sad to see you going through such a difficult experience" C. "If you tell me what's troubling you, I can help you solve your problems" D. "Suicide is a drastic solution to a problem that may not be such a serious matter"

A. "You must have been very upset when you tried to hurt yourself"

Which statement shows a nurse has empathy for a patient who made a suicide attempt? A. "You must have been very upset when you tried to hurt yourself." B. "It makes me sad to see you going through such a difficult experience." C. "If you tell me what is troubling you, I can help you solve your problems." D. "Suicide is a drastic solution to a problem that may not be such a serious matter."

A. "You must have been very upset when you tried to hurt yourself."

A nurse is caring for a client who lost a guardian to cancer last month. The client states, "I'd still have my guardian if the doctor would have made a diagnosis sooner." Which of the following responses should the nurse make? A. "You sound angry. Anger is a normal feeling associated with loss." B. "I think you would feel better if you talked about your feelings with a support group." C. "I understand just how you feel. I felt the same when my guardian died." D. "Do other members of your family also feel this way?"

A. "You sound angry. Anger is a normal feeling associated with loss."

A charge nurse is discussing the characteristics of a nurse-client relationship with a newly licensed nurse. Which of the following characteristics should the nurse include in the discussion? (SATA) A. A termination date is established B. An emotional commitment exists between the participants C. Behavioral change is encouraged D. The needs of both participants are met E. It's goal-directed

A. A termination date is established C. Behavioral change is encouraged E. It's goal-directed

An older adult was stopped by police while driving through a red light. When asked for a driver's license, the adult hands the police officer a pair of sunglasses. What sign of dementia is evident? A. Agnosia B. Aphasia C. Apraxia D. Memory impairment

A. Agnosia

What is your best intervention when you assess that a patient is responding to an auditory hallucination? A. Ask the patient, "Can you tell me what you are hearing?" B. Ask the patient, "Are you afraid of the voice you are hearing?" C. Tell the patient, "Try to ignore the voices you hear." D. Tell the patient, "The voices you hear are not real."

A. Ask the patient, "Can you tell me what you are hearing?"

A patient has progressive memory deficit associated with dementia. Which nursing intervention would best help the individual function in the environment? A. Assist the patient to perform simple tasks by giving step-by-step directions. B. Reduce frustration by performing activities of daily living for the patient. C. Stimulate intellectual function by discussing new topics with the patient. D. Promote the use of the patient's sense of humor by telling jokes.

A. Assist the patient to perform simple tasks by giving step-by-step directions.

A nurse is completing an admission assessment for a client who has schizophrenia. Which of the following findings should the nurse document as positive symptoms? (Select all that apply.) A. Auditory hallucination B. Lack of motivation C. Use of clang associations D. Delusion of persecution E. Constantly waving arms F. Flat affect

A. Auditory hallucination C. Use of clang associations D. Delusion of persecution E. Constantly waving arms

An appropriate intervention for a patient diagnosed with Bulimia nervosa who binges and purges is to teach the patient to: A. Avoid skipping meals/ restricting food B. Understand the value of reading journal entries aloud to others C. Eat a small meal after purging D. Concentrate oral intake after 4PM daily

A. Avoid skipping meals/ restricting food

Others describe a worker as very shy and lacking in self-confidence. This worker stays in an office cubicle all day and never comes out for breaks or lunch. Which term best describes this behavior? A. Avoidant B. Dependent C. Histrionic D. Paranoid

A. Avoidant

Which finding would prompt the nurse to carefully assess an 8-year-old child for development of a psychiatric disorder? A. Being raised by a parent with chronic major depressive disorder B. Moving to three new homes over a 2-year period C. Not being promoted to the next grade D. Having an imaginary friend

A. Being raised by a parent with chronic major depressive disorder

When counseling patients diagnosed with MDD, an ARNP will address the negative thought patterns by using: A. CBT B. Psychoanalytic therapy C. Alt. and complementary therapies D. Desensitization therapy

A. CBT

A client diagnosed with bipolar I disorder in the manic phase is yelling at another peer in the milieu. Which nursing intervention takes priority? A. Calmly redirect and remove the client from the milieu. B. Administer prescribed PRN intramuscular injection for agitation. C. Notify the client to lower voice. D. Obtain an order for seclusion to help decrease external stimuli.

A. Calmly redirect and remove the client from the milieu

A child diagnosed with attention deficit hyperactivity disorder (ADHD)is going to begin medication therapy. The nurse should plan to teach the family about which classification of medications? A. Central nervous system stimulants and non-stimulants B. Monoamine oxidase inhibitors (MAOIs) C. Antipsychotic medications D. Anxiolytic medications

A. Central nervous system stimulants and non-stimulants

Psychobiological agents showing promise for the treatment of cognitive impairment associated with AD include: A. Cholinesterase Inhibitors B. Herbals, including gingko biloba C. SSRIs and Trazodone D. Benzodiazepines and Buspirone

A. Cholinesterase Inhibitors

Psychobiological agents showing promise for the treatment of cognitive impairment associated with AD include: A. Cholinesterase inhibitors B. Herbals, including ginkgo biloba C. SSRIs and trazodone D. Benzodiazepines and buspirone

A. Cholinesterase inhibitors

A nurse is assisting with the development of protocols to address the increasing number of suicide attempts in the community. Which of the following interventions should the nurse include as a primary intervention? (Select all that apply.) A. Conducting a suicide risk screening on all new clients B. Creating a support group for family members of clients who completed suicide C. Educating high school teens about suicide prevention D. Initiating one‐on‐one observation for a client who has current suicidal ideation E. Teaching middle‐school educators about warning indicators of suicide

A. Conducting a suicide risk screening on all new clients C. Educating high school teens about suicide prevention E. Teaching middle‐school educators about warning indicators of suicide

A nurse assesses a confused older adult. The nurse experiences sadness and reflects. "The patient is like one of my grandparents, so helpless." What feelings does the nurse describe? A. Countertransference B. Catastrophic reaction C. Defensive coping reaction D. Transference

A. Countertransference

A nurse is caring for a client who smokes and has lung cancer. The client reports, "I'm coughing because I have that cold that everyone has been getting." The nurse should identify that the client is using which of the following defense mechanisms? A. Denial B. Reaction formation C. Sublimation D. Displacement

A. Denial

A charge nurse is preparing a staff education session on personality disorders. Which of the following personality characteristics associated with all of the personality disorders should the charge nurse include in the teaching? (Select all that apply.) A. Difficulty in getting along with other members of a group B. Belief in the ability to become invisible during times of stress C. Display of defense mechanisms when routines are changed D. Claiming to be more important than other persons E. Difficulty understanding why it is inappropriate to have a personal relationship with staff

A. Difficulty in getting along with other members of a group C. Display of defense mechanisms when routines are changed E. Difficulty understanding why it is inappropriate to have a personal relationship with staff

A nurse is planning care for the termination phase of a nurse-client relationship. Which of the following actions should the nurse include in the plan of care? A. Discussing ways to use new behaviors B. Establishing boundaries C. Developing goals D. Practicing new problem-solving skills

A. Discussing ways to use new behaviors

A nurse is working with a client who has recently lost a guardian. The nurse recognizes that which of the following factors influence a client's grief and coping ability? (Select all that apply.) A. Interpersonal relationships B. Culture C. Birth order D. Religious beliefs E. Prior experience with loss

A. Interpersonal relationships B. Culture D. Religious beliefs E. Prior experience with loss

Which of the following is least likely to contribute to building an effective therapeutic alliance between the nurse and a patient with anorexia? A. Establishing disciplined eating through the nurse's authoritarian approach with the patient B. Avoiding the stance of a parental role to foster a sense of empowerment C. Offering a highly structured approach in treating patients who are severely underweight D. Contracting with the outpatient person about treatment terms

A. Establishing disciplined eating through the nurse's authoritarian approach with the patient

A jitted college student is admitted to a hospital following a suicide attempt and states, "No one will ever love a loser like me." According to Erikson's theory of personality development, a nurse should recognize that this patient has a deficit in which developmental stage? A. Intimacy v Isolation B. Trust v mistrust C. Ego integrity v despair D. Initiative v guilt

A. Intimacy v Isolation

Chloe is now being seen by the ED physician. Her husband, Chad, is quietly demanding to see his wife. As the triage nurse, what are your best actions? (Select all that apply.) A. Have a staff member regularly touch base with Chad in the waiting room to reassure him that Chloe "is fine" but no room for visitors is provided. B. Immediately call hospital security. C. Move Chloe to secluded area in the ED so that you can interview her in private and advise her of safe shelters and offer brochures. D. Insist that Chloe admit she is being abused by Chad and immediately report the abuse to the police department.

A. Have a staff member regularly touch base with Chad in the waiting room to reassure him that Chloe "is fine" but no room for visitors is provided. C. Move Chloe to secluded area in the ED so that you can interview her in private and advise her of safe shelters and offer brochures.

A home health nurse is making a visit to a client who has Alzheimer's disease to assess the home for safety. Which of the following suggestions should the nurse make to decrease the client's risk for injury? A. Install extra locks at the top of exit doors. B. place rugs over electrical cords. C. put cleaning supplies on the top of a shelf. D. place the client's mattress on the floor. E. Install light fixtures above stairs.

A. Install extra locks at the top of exit doors. D. place the client's mattress on the floor. E. Install light fixtures above stairs.

Physical assessment of a patient diagnosed with Bulimia nervosa often reveals: A. Prominent parotid glands B. Amenorrhea C. Peripheral edema D. Thin, brittle hair

A. Prominent parotid glands

An older adult diagnosed with moderate-stage Alzheimer disease forgets where the bathroom is and has episodes of incontinence. Which intervention should the nurse suggest to the patient's family? A. Label the bathroom door. B. Take the older adult to the bathroom hourly. C. Place the older adult in disposable adult diapers. D. Make sure the older adult does not eat nonfood items.

A. Label the bathroom door.

Which cerebral structures should a nursing instructor describe to students as the "emotional brain"? A. Limbic system B. Left temporal lobe C. Cortex D. Cerebellum

A. Limbic system

What is likely to occur when a patient taking Lithium Carbonate has low Na+ lvls? A. Lithium toxicity B. Low serum Lithium lvls C. Increase in mania D. Decrease in mania

A. Lithium toxicity

A nurse is planning care for a child diagnosed with gender dysphoria. Which of the following nursing diagnoses could potentially document this client's problems? (Select all that apply.) A. Low self-esteem R/T rejection by peers B. Self-care deficit R/T isolative behaviors C. Disturbed personal identity R/T parenting patterns D. Impaired social interactions R/T socially unacceptable behaviors E. Activity intolerance R/T fatigue

A. Low self-esteem R/T rejection by peers C. Disturbed personal identity R/T parenting patterns D. Impaired social interactions R/T socially unacceptable behaviors

A client who was informed of the need for the emergency surgery. The client has a RR 30/min, and says, "This is difficult to comprehend. I feel shaky and nervous." The nurse should identify that the client is experiencing which of the following levels of anxiety? A. Moderate B. Severe C. Panic D. Panic E. Mild

A. Moderate

Which intervention takes priority when working with newly admitted clients experiencing suicidal ideations? A. Monitor the client at close, but irregular, intervals. B. Encourage the client to participate in group therapy. C. Enlist friends and family to assist the client to remain safe after discharge. D. Remind the client that it takes 4 to 6 weeks for antidepressants to be fully effective.

A. Monitor the client at close, but irregular, intervals

If an older adult patient must be physically restrained, who is responsible for the patient's safety? A. Nurse assigned to care for the patient B. Nursing assistant who applies the restraint C. Health care provider who ordered the application of the restraint D. Family member who agrees to the application of the restraint

A. Nurse assigned to care for the patient

A nurse is communicating with a client who was admitted for treatment of a substance use disorder. Which of the following communication techniques should the nurse identify as a barrier to therapeutic communication? A. Offering advice B. Reflecting C. Listening attentively D. Giving information

A. Offering advice

A nurse assessing a patient with suspected delirium will expect to find that the patient's symptoms developed: A. Over a period of hours to days B. Over a period of weeks to months C. With no relationship to another condition D. During middle age

A. Over a period of hours to days

A nurse is assessing a patient with suspected delirium will expect to find that the patient's symptoms developed: A. Over a period of hours to days B. Over a period of weeks to months C. With no relationship to another condition D. During middle age

A. Over a period of hours to days

A RN conducting group therapy on the eating disorders unit schedules the sessions immediately after meals for the primary purpose of: A. Processing the heighten anxiety associated with eating B. Maintaining patients' concentration and attention C. Shifting the patients' focus from food to psychotherapy D. Focusing on weight control mechanisms and food prep

A. Processing the heighten anxiety associated with eating

A patient diagnosed with moderate to severe Alzheimer disease has addressing and grooming self-care deficit. Designate the appropriate interventions to include in the patient's plan of care. (Select all that apply.) A. Provide clothing with elastic and hook-and-loop closures. B. Label clothing with the patient's name and name of the item. C. Administer antianxiety medication before bathing and dressing. D. Provide necessary items, and direct the patient to proceed independently. E. If the patient resists, use distraction and then try again after a short interval.

A. Provide clothing with elastic and hook-and-loop closures. B. Label clothing with the patient's name and name of the item. E. If the patient resists, use distraction and then try again after a short interval.

A mental health technician asks the nurse, "How do psychiatrists determine which diagnosis to give a patient?" Which of these responses by the nurse would be most accurate? A. Psychiatrists use pre-established criteria from the APA's DSM-5 B. Hospital policy dictates how psychiatrists diagnose mental disorders C. Psychiatrists assess the patient and identify diagnoses based on the patient's unhealthy responses and contributing factors D. The AMA identifies 10 diagnostic labels that psychiatrists can choose from

A. Psychiatrists use pre-established criteria from the APA's DSM-5

Which of the following matches the definition: the justification of behaviors using reason other than the real reason? A. Rationalization B. Projection C. Dysphoria D. Compensation

A. Rationalization

What is the priority intervention for a nurse beginning a therapeutic relationship with a patient diagnosed with a schizotypal personality disorder? A. Respect the patient's need for periods of social isolation. B. Prevent the patient from violating the nurse's rights. C. Engage the patient in many community activities. D. Teach the patient how to match clothing.

A. Respect the patient's need for periods of social isolation.

In caring for a patient with late AD, which nursing diagnosis demands the RN's highest priority? A. Risk for injury B. Self-care deficit C. Chronic low self-esteem D. Impaired verbal communication

A. Risk for injury

In caring for a patient with late AD, which nursing diagnosis demands the nurse's highest priority? A. Risk for injury B. Self-care deficit C. Chronic low self-esteem D. Impaired verbal communication

A. Risk for injury

An older adult, diagnosed with Alzheimer disease, lives with family and has multiple bruises. The home health nurse talks with the older adult's daughter, who becomes defensive and says, "My mother often wanders at night. Last night she fell down the stairs." Which nursing diagnosis has priority? A. Risk for injury, related to cognitive impairment and lack of caregiver supervision B. Noncompliance, related to confusion and disorientation as evidenced by lack of cooperation C. Impaired verbal communication, related to brain impairment as evidenced by the confusion D. Insomnia, related to cognitive impairment as evidenced by wandering at night

A. Risk for injury, related to cognitive impairment and lack of caregiver supervision

A patient diagnosed with Borderline PD has been hospitalized several times after self-inflicted lacerations. The patient remains impulsive. DBT starts on an outpatient basis. Which nursing diagnosis is focus of this therapy? A. Risk for self-mutilation B. Impaired skin integrity C. Powerlessness D. Risk for injury

A. Risk for self-mutilation

A patient diagnosed with borderline personality disorder has been hospitalized several times after self-inflicted lacerations. The patient remains impulsive. Dialectical behavior therapy starts on an outpatient basis. Which nursing diagnosis is the focus of this therapy? A. Risk for self-mutilation B. Impaired skin integrity C. Risk for injury D. Powerlessness

A. Risk for self-mutilation

Q1A patient at the emergency department is diagnosed with a concussion. The patient is accompanied by a spouse who insists on staying in the room and answering all questions. The patient avoids eye contact and has a sad affect and slumped shoulders. Assessment of which additional problem has priority? A. Risk of intimate partner violence B. Phobia of crowded places C. Migraine headaches D. Depressive symptoms

A. Risk of intimate partner violence

When a patient diagnosed with schizophrenia was discharged 6 months ago, haloperidol (Haldol) was prescribed. The patient now says, "I stopped taking those pills. They made me feel like a robot." What likely side effects did the patient experience? A. Sedation and muscle stiffness B. Sweating, nausea, and diarrhea C. Mild fever, sore throat, and skin rash D. Headache, watery eyes, and runny nose

A. Sedation and muscle stiffness

A nurse cares for a patient diagnosed with paraphilia. The nurse expects the health care provider may prescribe which type of medication to reduce paraphilic behaviors? A. Selective serotonin reuptake inhibitor (SSRI) B. Erectile dysfunction medication C. Atypical antipsychotic medication D. Mood stabilizer

A. Selective serotonin reuptake inhibitor (SSRI)

A 20-year-old male client is admitted to the psychiatric unit with a diagnosis of schizophrenia, acute episode. He is having auditory hallucinations and seems disoriented to time and place. The nurse knows that a hallucination can be explained as a(n) A. Sensory experience without foundation in reality. B. Distortion of real auditory or visual perception. C. Voice that is heard by the client but is not really there. D. Idea without foundation in reality.

A. Sensory experience without foundation in reality

Nursing staff that care for patients who are cognitively impaired can develop burnout. Strategies to avoid the development of burnout include: A. Setting realistic patient goals. B. Insulating self from emotional involvement with patients. C. Sedating patients to promote rest and minimize catastrophic episodes. D. Encouraging the family to permit the use of restraints to promote patient safety.

A. Setting realistic patient goals

A 50-year-old male client has a history of many hospitalizations for schizophrenia. He has been on long-term phenothiazines (Thorazine [chlorpromazine]), 400 mg/ day. The nurse assessing this client observes that he demonstrates erratic choreiform movements, lip smacking, and neck and back tonic contractions. From these symptoms and his history, the nurse concludes that the client has developed A. Tardive dyskinesia. B. Parkinsonism. C. Dystonia. D. Akathisia.

A. Tardive dyskinesia

A nurse works with a patient diagnosed with Schizo regarding the importance of med. management. The patient repeatedly says, "I don't like taking pills." Which treatment strategy should the RN discuss with the patient and HCP? A. Use of long-acting antipsychotic injections B. Adjunctive use of an antidepressant, like Amitriptyline (Elavil) C. Addition of a Benzodiazepine, like Lorazepam (Ativan) D. Inpatient hospitalization because of the high risk for exacerbation of symptoms

A. Use of long-acting antipsychotic injections

A nurse works with a patient diagnosed with schizophrenia regarding the importance of medication management. The patient repeatedly says, "I don't like taking pills." Which treatment strategy should the nurse discuss with the patient and health care provider? A. Use of long-acting antipsychotic injections B. Addition of a benzodiazepine, such as lorazepam (Ativan) C. Adjunctive use of an antidepressant, such as amitriptyline (Elavil) D. Inpatient hospitalization because of the high risk for exacerbation of symptoms

A. Use of long-acting antipsychotic injections

An older adult takes digoxin and hydrochlorothiazide daily, as well as lorazepam (Ativan) as needed for anxiety. Over 2 days, this adult developed confusion, slurred speech, an unsteady gait, and fluctuating levels of orientation. These findings are most characteristic of: A. delirium. B. dementia. C. amnestic syndrome. D. Alzheimer disease.

A. delirium.

A parent diagnosed with schizophrenia and her 13-year-old child live in a homeless shelter. The child has formed a trusting relationship with a shelter volunteer. The child says, "My three friends and I got an A on our school science project." The nurse can assess that the child: A. displays resiliency. B. has a difficult temperament. C. is at risk for posttraumatic stress disorder. D. uses intellectualization to deal with problems

A. displays resiliency.

A community mental health nurse plans an educational program for staff members at a home health agency that specializes in the care of older adults. A topic of high priority should be: A. identifying depression in older adults. B. providing cost-effective foot care for older adults. C. identifying nutritional deficiencies in older adults. D. psychosocial stimulation for those who live alone

A. identifying depression in older adults.

A 78-year-old nursing home resident diagnosed with hypertension and cardiac disease is usually alert and oriented. This morning, however, the resident says, "My family visited during the night. They stood by the bed and talked to me." In reality, the patient's family lives 200 miles away. The nurse should first suspect that the resident: A. may be experiencing side effects associated with medications. B. may be developing Alzheimer disease associated with advanced age. C. had a transient ischemic attack and developed sensory perceptual alterations. D. has previously unidentified alcohol abuse and is beginning alcohol withdrawal delirium.

A. may be experiencing side effects associated with medications.

A patient diagnosed with schizophrenia begins to talks about "cracklomers" in the local shopping mall. The term "cracklomers" should be documented as: A. neologism. B. concrete thinking. C. thought insertion. D. an idea of reference.

A. neologism

A nurse working on an acute mental health unit is admitting a client who has major depressive disorder and comorbid anxiety disorder. Which of the following actions is the nurse's priority? A. placing the client on one‐to‐one observation B. Assisting the client to perform ADLs C. Encouraging the client to participate in counseling D. Teaching the client about medication adverse effects

A. placing the client on one‐to‐one observation

A tricyclic antidepressant is prescribed for an older adult patient diagnosed with major depressive disorder. Nursing assessment should include careful collection of information regarding: A. use of other prescribed medications and over-the-counter products. B. evidence of pseudoparkinsonism or tardive dyskinesia. C. history of psoriasis and any other skin disorders. D. current immunization status.

A. use of other prescribed medications and over-the-counter products.

32) A nurse receives this laboratory result for a patient diagnosed with bipolar disorder: lithium level 1 mEq/L. This result is: A. within therapeutic limits. B. below therapeutic limits. C. above therapeutic limits. D. likely to be inaccurate.

A. within therapeutic limits

A nurse is providing teaching to the family of a client who has a substance use disorder. Which of the following statements by a family member indicates an understanding of the teaching? (Select all that apply.) A. "We need to understand that our sibling is responsible for their disorder." B. "Eliminating codependent behavior will promote recovery." C. "Our sibling should participate in an Al‐Anon group to assist with recovery." D. "The primary goal of treatment is abstinence from substance use." E. "Our sibling needs to discuss personal feelings about substance use to help with recovery."

B. "Eliminating codependent behavior will promote recovery." D. "The primary goal of treatment is abstinence from substance use." E. "Our sibling needs to discuss personal feelings about substance use to help with recovery."

A person diagnosed with schizophrenia has had difficulty keeping a job because of arguing with coworkers and accusing them of conspiracy. Today the person shouts, "They're all plotting to destroy me." Select the nurse's most therapeutic response. A. "Everyone here is trying to help you. No one wants to harm you." B. "Feeling that people want to destroy you must be very frightening." C. "That is not true. People here are trying to help if you will let them." D. "Staff members are health care professionals who are qualified to help you."

B. "Feeling that people want to destroy you must be very frightening."

A bipolar client states, "I am rich, and I feel I must give money to you." Which response should the nurse make? A. "Why do you think you feel the need to give money away?" B. "I am here to provide care and cannot accept this from you." C. "I can request that your case manager discuss appropriate charity options with you." D. "You should know that giving away your money is inappropriate."

B. "I am here to provide care and cannot accept this from you."

A nurse is caring for a client who has schizoaffective disorder. Which of the following statements indicates the client is experiencing depersonalization? A. "I am a superhero and am immortal." B. "I am no one, and everyone is me." C. "I feel monsters pinching me all over." D. "I know that you are stealing my thoughts."

B. "I am no one, and everyone is me."

A nurse is providing the teaching for a client who is scheduled to receive ECT for the treatment of MDD. Which of the following client statements indicates understanding of the teaching? A. "It's common to treat depression with ECT before trying medications" B. "I will receive a muscle relaxant to protect me from injury during ECT" C. "I can have my depression cured if I receive a series of ECT treatments" D. "I should receive ECT once a week for 6 weeks"

B. "I will receive a muscle relaxant to protect me from injury during ECT"

A patient diagnosed with Schizo tells the RN, "I eat skiller. Tend to end. Easter. It blows away. Get it?" Select the RN's best response: A. "Your thoughts are very disconnected." B. "I'm having difficulty understanding what you're saying." C. "Try to organize your thoughts, and then tell me again." D. "Nothing you are saying is clear."

B. "I'm having difficulty understanding what you're saying."

An 80-year-old patient has difficulty walking because of arthritis and says, "It's awful to be old. Every day is a struggle. No one cares about old people." Which is the nurse's most therapeutic response? A. "Everyone here cares about old people. That's why we work here." B. "It sounds like you're having a difficult time. Tell me about it." C. "Let's not focus on the negative. Tell me something good." D. "You are still able to get around, and your mind is alert."

B. "It sounds like you're having a difficult time. Tell me about it."

Jessie had a blood alcohol level (BAL) of 0.11% upon arrival at the emergency department (ED). She is now your patient in the hospital psychiatric unit, day 4. She tells you, "I wasn't drunk. I just had a few beers." What is an appropriate response? A. "Jessie, of course you were drunk. You always are." B. "Jessie, your BAL was 0.11%. That clearly indicates that you had alcohol intoxication." C. "You are in denial, and that will impede your recovery from alcoholism." D. "Until you recognize your problem, you will never win over this addiction."

B. "Jessie, your BAL was 0.11%. That clearly indicates that you had alcohol intoxication."

The most appropriate short-term nursing goal for clients with schizophrenia is to A. Set limits on bizarre behavior. B. Establish a trusting, nonthreatening relationship. C. Quickly establish a warm, close relationship. D. Protect client from inappropriate impulses.

B. Establish a trusting, nonthreatening relationship

A nurse is teaching a client who has an anxiety disorder and is scheduled to begin classical psychoanalysis. Which of the following client statements indicates an understanding of this form of therapy? A."Psychoanalysis will help me reduce my anxiety by changing my behaviors." B. "The therapist will focus on my past relationships during our sessions." C. "Even if my anxiety improves, I will need to continue this therapy for 6 weeks." D. "This therapy will address my conscious feelings about stressful experiences."

B. "The therapist will focus on my past relationships during our sessions."

A client began taking lithium for the treatment of bipolar disorder approx. 1 month ago and asks why he has gained 12 lbs since then. Which is the most appropriate nursing response? A. "Your weight gain is more likely r/t food intake with the medication" B. "Weight gain is common, but troubling side effect. Let's talk about some strategies for safely improving your nutrition and exercise habits" C. "There's not much you can do about the weight gain, it's better than being emotionally unstable though" D. "I'm surprised you have gained, weight loss is the typical pattern when taking lithium"

B. "Weight gain is common, but troubling side effect. Let's talk about some strategies for safely improving your nutrition and exercise habits

A patient with a diagnosis of major depression who has attempted suicide says to the nurse, "I should have died! I've always been a failure. Nothing ever goes right for me." Which response demonstrates therapeutic communication? A. "You have everything to live for." B. "Why do you see yourself as a failure?" C. "Feeling like this is all part of being depressed." D. "You've been feeling like a failure for a while?"

B. "Why do you see yourself as a failure?"

Your patient asks, "Will Antabuse really help me with my drinking problem?" What is your most appropriate response? A. "Yes, if you are motivated." B. "Yes, if you use it correctly." C. "That is totally up to you." D. "Antabuse works well for some."

B. "Yes, if you use it correctly."

A nurse is caring for a client who has early stage Alzheimer's disease and a new prescription for donepezil. The nurse should include which of the following statements when teaching the client about the medication? A. "You should avoid taking over‐the‐counter acetaminophen while on donepezil." B. "You should take this medication before going to bed at the end of the day." C. "You will be screened for underlying kidney disease prior to starting donepezil." D. "You should stop taking donepezil if you experience nausea or diarrhea."

B. "You should take this medication before going to bed at the end of the day."

A nurse is planning care for several clients who are attending community-based mental health programs. Which of the following clients should the nurse visit first? A. A client who tells the nurse about experiencing manifestations of severe anxiety before and during a job interview B. A client who reports hearing a voice saying that life is not worth living anymore C. A client who received a burn on the arm while using a hot iron at home D. A client who requests a change of antipsychotics due to some new adverse effects

B. A client who reports hearing a voice saying that life is not worth living anymore

Which statement about crisis theory provides a basis for nursing intervention? A. Crisis is precipitated by an event that enhances the person's self-concept and self-esteem. B. A crisis is an acute, time-limited phenomenon experiences as an overwhelming emotional reaction to a problem perceived as unsolvable. C. A person in crisis usually has had adjustment problems and has inadequately coped in their usual life situations. D. Nursing intervention in crisis situations rarely has the effect of ameliorating the crisis.

B. A crisis is an acute, time-limited phenomenon experiences as an overwhelming emotional reaction to a problem perceived as unsolvable.

Which situation reflects a violation of the ethical principle of veracity? A. A nurse does not treat all of the clients equally, regardless of illness severity B. A nurse tricks a client into seclusion by asking the client to carry linen to the seclusion room C. A nurse discusses with a client another client's impending discharge D. A nurse refuses to give information to a physician who's not responsible for the client's care

B. A nurse tricks a client into seclusion by asking the client to carry linen to the seclusion room

Which nursing intervention demonstrates false imprisonment? A. An involuntary hospitalized patient with suicidal ideation attempts to leave the unit. A nurse calls the security team and uses established protocols to prevent the patient from leaving B. A patient has been imitating, seeking attention of nurses most of the day. Now a nurse escorts the patient down the hall, saying, "Stay in your room or you'll be put in seclusion." C. An involuntarily hospitalized patient with suicidal ideation runs out of the psych unit. A nurse rushes after the patient and convinces the patient to return to the unit D. A confused and combative patient says, "I'm getting out of here and no one can stope me." The nurse restrains this patient without an order and then promptly obtains an order

B. A patient has been imitating, seeking attention of nurses most of the day. Now a nurse escorts the patient down the hall, saying, "Stay in your room or you'll be put in seclusion."

An adult with paranoia becomes agitated and threatens to assault a staff person. Select the best initial nursing intervention. A. Tell the patient, "If you do not calm down, seclusion will be needed." B. Address the patient with simple directions and a calming voice. C. Help the patient focus by rubbing the patient's shoulders. D. Offer the patient a dose of antipsychotic medication. E. Reorient the patient to the time and place.

B. Address the patient with simple directions and a calming voice.

Hours after an apartment fire in which all the family's household goods and clothing were lost. Ms. T has no family in the area. Her efforts to mobilize assistance have been disorganized, and she is still without shelter. She is distraught and confused. The nurse assesses the situation as which of the following types of crisis? A. Situational B. Adventitious C. Evidence of an inadequate personality D. Maturational

B. Adventitious

A patient says to the RN, "My life doesn't have happiness in it anymore. I once enjoyed holidays, but now they're just another day." How would the RN document the complaint? A. Euphoria B. Anhedonia C. Anergia D. Vegetative

B. Anhedonia

A patient says to the nurse, "My life does not have any happiness in it anymore. I once enjoyed holidays, but now they're just another day." How would the nurse document the complaint? A. Vegetative B. Anhedonia C. Euphoria D. Anergia

B. Anhedonia

A nurse is speaking with a client who has schizophrenia when the client suddenly seems to stop focusing on the nurse's questions and begins looking at the ceiling and talking to themselves. Which of the following actions should the nurse take? A. Stop the interview at this point and resume later when the client is better able to concentrate. B. Ask the client, "Are you seeing something on the ceiling?" C. Tell the client, "You seem to be looking at something on the ceiling. I see something there, too." D. Continue the interview without comment on the client's behavior.

B. Ask the client, "Are you seeing something on the ceiling?"

What's your best intervention when you assess that a patient is responding to an auditory hallucination? A. Tell the patient, "Try to ignore the voices you hear." B. Ask the patient, "Can you tell me what you're hearing?" C. Tell the patient, "The voices you hear are not real." D. Ask the patient, "Are you afraid of the voice you're hearing?"

B. Ask the patient, "Can you tell me what you're hearing?"

Nancy, age 23, has just been hospitalized after she reported to her college roommate, Carol, that she had swallowed a bottle of aspirin. She has been stabilized in the ER and has been admitted to the psychiatric unit. This is Nancy's third hospitalization for similar behavior since age 15. Nancy reports that her parents were divorced when she was 3 years old. She has not seen her father since that time. She lived with her mother until she was 6 years old, and then her mother left her with her grandmother and went to "seek fame and fortune" in Hollywood. She sees her mother rarely. Nancy's roommate tells the admitting nurse that Nancy has been a "nervous" person ever since she has known her. "She has mood swings, and the smallest things will set her off. Once when she was particularly 'down,' I walked in on her sitting on her bed. It looked like she was cutting her arm with a razor blade. When I confronted her about it ,she denied it. But I saw the blood. It really freaked me out!" Nancy's boyfriend recently broke up with her. Nancy has had a succession of boyfriends since high school. All of her relationships are very intense, and she becomes hysterical and then despondent when a boy bre

B. Borderline

A nurse plans the care for an individual diagnosed with antisocial personality disorder. Which characteristic behaviors will the nurse expect? (Select all that apply.) A. Reclusive behavior B. Callous attitude C. Perfectionism D. Aggression E. Clinginess F. Anxiety

B. Callous attitude D. Aggression

A patient diagnosed with schizophrenia believes evil spirits are being summoned by a local minister and verbally threatens to bomb a local church. The psychiatrist notifies the minister. The psychiatrist has: A. Avoided charges of malpractice B. Demonstrated the duty to warn and protect C. Released information without proper authorization D. Violated the patient's confidentiality

B. Demonstrated the duty to warn and protect

A patient admitted to an alcoholism rehab program says, "I'm just a social drinker, I usually have a drink or 2 at brunch, a few cocktails in the afternoon, wine at dinner, and several drinks during the evening." The patient is using which defense mechanism? A. Introjection B. Denial C. Rationalization D. Projection

B. Denial

A charge nurse is reviewing Kübler‐Ross: Five Stages of Grief with a group of newly licensed nurses. Which of the following stages should the charge nurse include in the teaching? (Select all that apply.) A. Disequilibrium B. Denial C. Bargaining D. Anger E. Depression

B. Denial C. Bargaining D. Anger E. Depression

A nurse in the emergency department tells an adult, "Your mother had a serious stroke." The adult tearfully says, "Who will take care of me now? My mother always told me what to do, what to wear, and what to eat. I need someone to reassure me when I get anxious." Which term best describes this behavior? A. Histrionic B. Dependent C. Narcissistic D. Borderline

B. Dependent

A nurse is discussing relapse prevention with a bipolar pt. Which information should the nurse teach? (Select all that apply.) A. Use caffeine in moderation to prevent relapse. B. Difficulty sleeping can indicate a relapse. C. Begin taking your medications as soon as a relapse begins. D. participating in psychotherapy can help prevent a relapse. E. Anhedonia is a clinical manifestation of a depressive relapse.

B. Difficulty sleeping can indicate a relapse D. participating in psychotherapy can help prevent a relapse E. Anhedonia is a clinical manifestation of a depressive relapse

For the third time within a month, a client with borderline personality disorder took a handful of pills, called 911, and was admitted to the emergency department. The nurse overhears an unlicensed staff member say, "Here she comes again. If she was serious about committing suicide, she'd have done it by now." What should the nurse say? A. Clients with personality disorders rarely have completed suicides. B. Each suicidal attempt should be taken seriously. C. Exploration of suicidal ideation and intent should be avoided. D. The nurse should prepare the client for direct inpatient admission.

B. Each suicidal attempt should be taken seriously

A nurse decides to put a client who has a psychotic disorder in seclusion overnight because the unit is very short-staffed, and the client frequently fights with other clients. The nurse's actions are an example of which of the following torts? A. Invasion of privacy B. False imprisonment C. Assault D. Battery

B. False imprisonment

A nurse is performing an admission assessment for a client who has delirium related to an acute urinary tract infection. Which of the following findings should the nurse expect? (Select all that apply.) A. History of gradual memory loss B. Family report of personality changes C. Hallucinations D. Unaltered level of consciousness E. Restlessness

B. Family report of personality changes C. Hallucinations E. Restlessness

A nurse is assessing a pt experiencing alcohol withdrawal. Which findings should the nurse expect? (Select all that apply.) A. Bradycardia B. Fine tremors of both hands C. Hypotension D. Vomiting E. Restlessness

B. Fine tremors of both hands D. Vomiting E. Restlessness

When preparing to interview a patient diagnosed with narcissistic PD, a RN can anticipate the assessment findings will include: A. Charm, drama, seductiveness; seeking admiration B. Grandiosity, attention seeking, and arrogance C. Preoccupation with minute details; perfectionism D. Difficulty being alone; indecisiveness, submissiveness

B. Grandiosity, attention seeking, and arrogance

A nurse is caring for a pt who has MDD. Which are risk factors for depression? (Select all that apply.) A. Male sex B. History of chronic bronchitis C. Recent death in client's family D. Family history of depression E. personal history of panic disorder

B. History of chronic bronchitis C. Recent death in client's family D. Family history of depression E. personal history of panic disorder

A patient experiencing acute mania has exhausted staff members by noon. The patient joked, manipulated, insulted, and fought all morning. Staff members are feeling defensive and fatigued. Which is best action? A. Confer with HCP regarding use of seclusion B. Hold staff meeting to discuss consistency and limit setting approaches C. Conduct a meeting with all patients to discuss behavior D. Explain to patient that the behavior is unacceptable

B. Hold staff meeting to discuss consistency and limit setting approaches

A patient with bulimia nervosa has become dehydrated from self-induced vomiting. This is most likely to result in: A. Hyperchloremia. B. Hypokalemia. C. Tachycardia. D. Parotid gland atrophy.

B. Hypokalemia

A nurse is performing an admission assessment of a client who has bulimia nervosa with purging behavior. Which of the following is an expected finding? (Select all that apply.) A. Amenorrhea B. Hypokalemia C. Yellowing of the skin D. Slightly elevated body weight E. presence of lanugo on the face

B. Hypokalemia D. Slightly elevated body weight

A child is diagnosed with Tourette's disorder and is withdrawn, refuses to participate in group therapy, and initiates little or no communication with peers. On the basis of the symptoms presented, which nursing diagnosis should the nurse assign to this child? A. Risk for self-directed or other-directive violence B. Impaired social interaction C. Low self-esteem D. Anxiety (severe)

B. Impaired social interaction

A nurse is caring for a client on an acute mental health unit. The client reports hearing voices that are stating, "kill your doctor." Which of the following actions should the nurse take first? A. Encourage the client to participate in group therapy on the unit. B. Initiate one‐to‐one observation of the client. C. Focus the client on reality. D. Notify the provider of the client's statement.

B. Initiate one‐to‐one observation of the client

A nurse is caring for a client who states, "I plan to commit suicide." Which of the following assessments should the nurse identify as the priority? A. Client's educational and economic background B. Lethality of the method and availability of means C. Quality of the client's social support D. Client's insight into the reasons for the decision

B. Lethality of the method and availability of means

A patient diagnosed with Schizo begins talks about "cracklomers" in the local shopping mall. The term "cracklomers" should be documented as: A. An idea of reference B. Neologism C. Concrete thinking D. Thought insertion

B. Neologism

A nurse is planning care for a client who has bipolar disorder and is experiencing a manic episode. Which of the following interventions should the nurse include in the plan of care? (Select all that apply.) A. Provide flexible client behavior expectations. B. Offer concise explanations. C. Establish consistent limits. D. Disregard client concerns. E. Use a firm approach with communication.

B. Offer concise explanations C. Establish consistent limits E. Use a firm approach with communication

A 53-year-old man who has been on your psychiatric unit for 3 weeks is calm and controlled and socializes with other patients. He goes to off-unit recreation. He is diagnosed with major depressive disorder (MDD). You overhear the patient tell another patient that he wants to kill himself. He shows no signs of wanting to hurt himself and continues to socialize. You confront the patient, and he says it was just a joke and he has no desire to kill himself. What do you do first? A. Ignore the comment as a meaningless statement. B. Place the patient on constant observation. C. Call the practitioner to assess the patient. D. Move the patient's room closer to the nurse's station.

B. Place the patient on constant observation

A patient has taken trifluoperazine (Stelazine) 30 mg/day orally for 3 years. The clinic nurse notes that the patient grimaces and constantly smacks both lips. The patient's neck and shoulders twist in a slow, snakelike motion. Which problem would the nurse suspect? A. Agranulocytosis B. Tardive dyskinesia C. Tourette syndrome D. Anticholinergic effects

B. Tardive dyskinesia

The nurse expects to establish a supportive therapeutic relationship with a client diagnosed with schizotypal personality disorder. Which nursing intervention is most appropriate? A. Set limits on acting-out behaviors and explain consequences. B. Present reality when client is experiencing magical thinking. C. Encourage client to gradually verbalize hostile feelings. D. Remove all dangerous objects from the environment

B. Present reality when client is experiencing magical thinking

A client with the diagnosis of schizophrenia has orders for Clozaril (Clozapine). The nurse will evaluate the drug's effect as positive if the A. Client's energy level and involvement in activities goes up. B. Psychotic symptoms, like hearing voices, are reduced. C. Monthly liver function studies change moderately. D. A client develops leukopenia.

B. Psychotic symptoms, like hearing voices, are reduced.

Which ability should the nurse expect from a client in the mild stage of dementia of the Alzheimer's type? A. Remembering the daily schedule. B. Recalling past events. C. Coping the anxiety. D. Solving problems of daily living.

B. Recalling past events

What is a nurse's legal responsibility if child abuse or neglect is suspected? A. Discuss the findings with the child's teacher, principal, and school psychologist. B. Report the suspected abuse or neglect according to state regulations. C. Document the observations and speculations in the medical record. D. Continue the assessment.

B. Report the suspected abuse or neglect according to state regulations.

What is the priority nursing diagnosis for a patient experiencing fluctuating levels of consciousness, disturbed orientation, and visual and tactile hallucinations? A. Bathing/hygiene self-care deficit related to altered cerebral function as evidenced by confusion and inability to perform personal hygiene tasks B. Risk for injury related to altered cerebral function, misperception of the environment, and unsteady gait C. Disturbed thought processes related to medication intoxication as evidenced by confusion, disorientation, and hallucinations D. Fear related to sensory perceptual alterations as evidenced by hiding from imagined ferocious dogs

B. Risk for injury related to altered cerebral function, misperception of the environment, and unsteady gait

A person intentionally OD's on antidepressant drugs. Which nursing diagnosis has highest priority? A. Social isolation B. Risk for suicide C. Ineffective management of the therapeutic regimen D. Powerlessness

B. Risk for suicide

A patient diagnosed with Borderline PD has a hx of self-mutilation and suicide attempts. The patient reveals feelings of depression and anger with life. The psychiatrist suggest the use of a med. Which type of med should the RN expect? A. Antipsychotic B. SSRI C. MAOI D. Benzodiazepine

B. SSRI

A nurse should expect that an increase in dopamine activity might play a significant role in the development of which mental illness? A. Body dysmorphic disorder B. Schizophrenia spectrum disorder C. Major depressive disorder D. Generalized anxiety disorder

B. Schizophrenia spectrum disorder

The cause of Bipolar disorder hasn't been determined, but: A. Excess NE is probably a major factor B. Several factors, including genetics, are implicated C. Brain structures were altered by trauma early in life D. Excess sensitivity in DA receptors may exist

B. Several factors, including genetics, are implicated

A nurse is assessing a client who has generalized anxiety disorder. Which of the following findings should the nurse expect? (SATA) A. Impulsive decision making B. Sleep disturbance C. Restlessness D. Excessive worry for 6 months E. Delayed reflexes

B. Sleep disturbance C. Restlessness D. Excessive worry for 6 months

The parent of a child who is newly diagnosed with autism disorder asks the psychiatric nurse how the parent should interact with the child. What is the psychiatric nurse's best response? A. Ask your pediatrician. B. Speak with the child face-to-face, maintain a safe environment, give rewards for good behaviors, and do not punish for bad behaviors, which may lead to self-injury by the child. C. Treat the child as you would treat other children. D. Send the parent to an autism disorder website.

B. Speak with the child face-to-face, maintain a safe environment, give rewards for good behaviors, and do not punish for bad behaviors, which may lead to self-injury by the child.

A nurse is caring for a client who has borderline personality disorder. The client says, "The nurse on the evening shift is always nice! You are the meanest nurse ever!" The nurse should recognize the client's statement as an example of which of the following defense mechanisms? A. Regression B. Splitting C. Undoing D. Identification

B. Splitting

A nurse is assessing a pt with pedophilic disorder. What would differentiate this from sexual dysfunction? A. Symptoms of sexual dysfunction include inappropriate sexual behaviors, whereas symptoms of a sexual disorder include impairment in normal sexual response. B. Symptoms of a sexual disorder include inappropriate sexual behaviors, whereas symptoms of sexual dysfunction include impairment in normal sexual response. C. Sexual dysfunction can be caused by increased levels of circulating androgens, whereas levels of circulating androgens do not affect sexual disorders. D. Sexual disorders can be caused by decreased levels of circulating androgens, whereas levels of circulating androgens do not affect sexual dysfunction.

B. Symptoms of a sexual disorder include inappropriate sexual behaviors, whereas symptoms of sexual dysfunction include impairment in normal sexual response

A 50 y/o male client has a hx of many hospitalizations for schizophrenia. He's been on long-term phenothiazines 400mg/day/ The nurse assessing this client observes that he demonstrates erratic choreiform movements, lip smacking, and neck and back tonic contractions. From these symptoms and his hx, the RN concludes that the client has developed: A. Dystonia B. Tardive dyskinesia C. Akathisia D. Parkinsonism

B. Tardive dyskinesia

A 23-year-old mother asks the psychiatric nurse for the telephone number of a child psychiatrist because her 2-year-old daughter constantly refuses to do what she is told to do. She throws a tantrum in public when she does not get her way. The mother wants the psychiatrist to treat her daughter for oppositional defiant disorder (ODD). What is the best response by the psychiatric nurse? A. Give the mother names and telephone numbers of a few local psychiatrists. B. Tell the mother that the child is demonstrating normal behavior and her actions are consistent with a child struggling with emerging independence. C. Tell the mother that treatment for ODD does not begin until age 3. D. Tell the mother to take the child to her pediatrician to rule out underlying medical conditions that might be causing the problem.

B. Tell the mother that the child is demonstrating normal behavior and her actions are consistent with a child struggling with emerging independence.

A nurse hears a newly licensed nurse discussing a client's hallucinations in the hallway with another nurse. Which of the following actions should the nurse take first? A. Notify the nurse manager. B. Tell the nurse to stop discussing the behavior. C. Provide an in-service program about confidentiality. D. Complete an incident report.

B. Tell the nurse to stop discussing the behavior.

A nurse is collecting an admission history for a client who has acute stress disorder (ASD). Which of the following client behaviors should the nurse expect? A. The client remembers many details about the traumatic incident B. The client expresses a sense of unreality about the traumatic incident C. The client remembers first noticing manifestations of the disorder 6 weeks after the traumatic incident occurred D. The client expresses heightened elation about what's happening

B. The client expresses a sense of unreality about the traumatic incident

Which statement about TCAs is accurate? A. They may cause hypomania or recent memory impairment B. Their full therapeutic potential may not be reached until 4 weeks C. They should not be given with antianxiety agents D. Strong or aged cheese should not be eaten while taking them

B. Their full therapeutic potential may not be reached until 4 weeks

At what point should the RN determine that a client is at risk for developing a mental illness? A. When thoughts, feelings, and behaviors aren't reflective of DSM-5 criteria B. When maladaptive responses to stress are coupled with interference in daily functioning C. When a client communicates significant distress D. When a client uses defense mechanisms as ego protection

B. When maladaptive responses to stress are coupled with interference in daily functioning

A RN receives this lab result for a patient diagnosed with Bipolar disorder: Lithium level 1mEq/L. This result is: A. Below therapeutic limits B. Within therapeutic limits C. Likely to be inaccurate D. Above therapeutic limits

B. Within therapeutic limits

A newly admitted patient diagnosed with schizophrenia is hypervigilant and constantly scans the environment. The patient states, "I saw two doctors talking in the hall. They were plotting to kill me." The nurse may correctly assess this behavior as: A. echolalia. B. an idea of reference. C. a delusion of infidelity. D. an auditory hallucination.

B. an idea of reference

A priority nursing intervention for a patient diagnosed with major depressive disorder is: A. distracting the patient from self-absorption. B. carefully and inconspicuously observing the patient around the clock. C. allowing the patient to spend long periods alone in self-reflection. D. offering opportunities for the patient to assume a leadership role in the therapeutic milieu.

B. carefully and inconspicuously observing the patient around the clock

Irresponsible behavior is to a pt with cluster B personality disorder as avoidant behavior is to a client diagnosed with a: A. Cluster A personality disorder. B. Cluster B personality disorder. C. Cluster C personality disorder. D. Cluster D personality disorder.

C. Cluster C personality disorder

This nursing diagnosis applies to a patient experiencing mania: imbalanced nutrition: less than body requirements, related to insufficient caloric intake and hyperactivity as evidenced by 5-pound weight loss in 4 days. Select the most appropriate outcome. The patient will: A. ask staff for assistance with feeding within 4 days. B. drink six servings of a high-calorie, high-protein drink each day. C. consistently sit with others for at least 30 minutes at mealtime within 1 week. D. consistently wear appropriate attire for age and sex within 1 week while in the psychiatric unit.

B. drink six servings of a high-calorie, high-protein drink each day

A patient who has been diagnosed with schizoid personality disorder is newly admitted to the unit. The best initial nursing intervention is to: A. set firm limits. B. engage in trust building. C. involve in milieu and group activities. D. encourage identification and expression of feelings.

B. engage in trust building

When assessing a 2-year-old diagnosed with autism spectrum disorder, a nurse expects: A. hyperactivity and attention deficits. B. failure to develop interpersonal skills. C. history of disobedience and destructive acts. D. high levels of anxiety when separated from a parent.

B. failure to develop interpersonal skills.

The health care provider prescribes medication for a child diagnosed with attention deficit hyperactivity disorder (ADHD). The desired behavior for which the nurse should monitor is: A. increased expressiveness in communicating with others. B. improved ability for cooperative play with other children. C. ability to identify anxiety and implement self-control strategies. D. improved socialization skills with other children and authority figures.

B. improved ability for cooperative play with other children.

A patient hospitalized with a mood disorder has aggression, agitation, talkativeness, and irritability. A nurse begins the care plan based on the expectation that the health care provider is most likely to prescribe a medication classified as a(n): A. anticholinergic. B. mood stabilizer C. psychostimulant. D. tricyclic antidepressant.

B. mood stabilizer

A patient comes to the hospital for treatment of injuries sustained during a rape. The patient abruptly decides to decline treatment and return home. Before the patient leaves, the nurse should: A. tell the patient, "You may not leave until you receive prophylactic treatment for sexually transmitted diseases." B. provide written information concerning the physical and emotional reactions that may be experienced. C. explain the need and importance of human immunodeficiency virus(HIV) testing. D. offer verbal information about legal resources.

B. provide written information concerning the physical and emotional reactions that may be experienced.

A patient diagnosed with major depressive disorder begins selective serotonin reuptake inhibitor (SSRI) antidepressant therapy. Priority information given to the patient and family should include a directive to: A. avoid exposure to bright sunlight. B. report increased suicidal thoughts. C. restrict sodium intake to 1 g daily. D. maintain a tyramine-free diet.

B. report increased suicidal thoughts

A nurse set limits for a patient diagnosed with a borderline personality disorder. The patient tells the nurse, "You used to care about me. I thought you were wonderful. Now I can see I was mistaken. You're terrible." This outburst can be assessed as: A. denial. B. splitting. C. reaction formation. D. separation-individuation strategies.

B. splitting

During morning care, a nursing assistant asks a patient diagnosed with Dementia, "How was your night?" The patient replies, "It was lovely. I went out to dinner and a movie with my friend." Which term applies to the patient's response? A. Delirium B. Sundown syndrome C. Confabulation D. Perseveration

C. Confabulation

A nurse is caring for a client who is in mechanical restraints. Which of the following statements should the nurse include in the documentation? (SATA) A. "Client acted out after lunch" B. "Client ate most of their breakfast" C. "Client offered 8 oz of water every hr" D. "Client shouted obscenities at assistive personnel" E. "Client received chlorpromazine 15 mg PO at 1000"

C. "Client offered 8 oz of water every hr" D. "Client shouted obscenities at assistive personnel" E. "Client received chlorpromazine 15 mg PO at 1000"

A nurse is teaching a newly licensed nurse about the use of electroconvulsive therapy (ECT) for the treatment of bipolar disorder. Which of the following statements by the newly licensed nurse indicates understanding? A. "ECT is the recommended initial treatment for bipolar disorder." B. "ECT is contraindicated for clients who have suicidal ideation." C. "ECT is effective for clients who are experiencing severe mania." D. "ECT is prescribed to prevent relapse of bipolar disorder."

C. "ECT is effective for clients who are experiencing severe mania."

A person diagnosed with Schizo has had difficulty keeping a job because of arguing with coworkers and accusing them of conspiracy. Today the person shouts, "They're all plotting to destroy me." Select the RN's most therapeutic response: A. "Everyone here is trying to help you. No one wants to harm you." B. "That's not true. Ppl here are trying to help you if you'll let them." C. "Feeling that ppl want to destroy you must be very frightening." D. "Staff members are health care professionals who are qualified to help you."

C. "Feeling that ppl want to destroy you must be very frightening."

Which statement by the client indicates understanding of the teaching about premenstrual dysphoric disorder? A. "I can expect my problems with pMDD to be worst when I'm menstruating." B. "I should avoid exercising when I am feeling depressed." C. "I am aware that my pMDD causes me to have rapid mood swings." D. "I should increase my caloric intake with a nutritional supplement when my pMDD is active."

C. "I am aware that my pMDD causes me to have rapid mood swings."

A nurse manager is discussing the care of a client who has a personality disorder with a newly licensed nurse. Which of the following statements by the newly licensed nurse indicates an understanding of the teaching? A. "I can promote my client's sense of control by establishing a schedule." B. "I should encourage clients who have a schizoid personality disorder to increase socialization." C. "I should practice limit‐setting to help prevent client manipulation." D. "I should implement assertiveness training with clients who have antisocial personality disorder."

C. "I should practice limit‐setting to help prevent client manipulation."

A nurse is caring for a client who has bulimia nervosa and has stopped purging behavior. The client tells the nurse about fears of gaining weight. Which of the following responses should the nurse make? A. "Many clients are concerned about their weight. However, the dietitian will ensure that you don't get too many calories in your diet." B. "Instead of worrying about your weight, try to focus on other problems at this time." C. "I understand you have concerns about your weight, but first, let's talk about your recent accomplishments." D. "You are not overweight, and the staff will ensure that you do not gain weight while you are in the hospital. We know that is important to you."

C. "I understand you have concerns about your weight, but first, let's talk about your recent accomplishments."

A nurse is talking with the caregiver of a child who has demonstrated recent changes in behavior and mood. When the caregiver of the child asks the nurse about their child's condition, which of the following responses should the nurse make? A. "I'm not sure what's wrong. Have you asked the doctor about your concern?" B. "I'm sure everything will be okay, it just takes time to heal" C. "I understand you're concerned. Let's discuss what concerns you specifically" D. "I think your child is getting better. What have you noticed?"

C. "I understand you're concerned. Let's discuss what concerns you specifically"

A newly admitted patient diagnosed with schizophrenia says, "The voices are bothering me. They yell and tell me I'm bad. I have got to get away from them." Select the nurse's most helpful reply. A. "Do you hear the voices often?" B. "Do you have a plan for getting away from the voices?" C. "I will stay with you. Focus on what we are talking about, not the voices." D. "Forget about the voices. Ask some other patients to sit and talk with you."

C. "I will stay with you. Focus on what we are talking about, not the voices."

A charge nurse is discussing the care of a client who has major depressive disorder (MDD) with a newly licensed nurse. Which of the following statements by the newly licensed nurse indicates an understanding of the teaching? A. "Care during the continuation phase focuses on treating continued manifestations of MDD." B. "The treatment of MDD during the maintenance phase lasts for 6 to 12 weeks." C. "The client is at greatest risk for suicide during the first weeks of an MDD episode." D. "Medication and psychotherapy are most effective during the acute phase of MDD."

C. "The client is at greatest risk for suicide during the first weeks of an MDD episode."

A nurse is orienting a new client to a mental health unit. When explaining the unit's community meetings. which of the following statements should the nurse make? A. "Community meetings have a specific agenda that is established by staff." B. "You and a group of other clients will meet to discuss your treatment plans." C. "You and the other clients will meet with staff to discuss common probs." D. "Community meetings are an excellent opportunity to explore your personal mental health issues."

C. "You and the other clients will meet with staff to discuss common probs."

A client with the diagnosis of paranoid personality disorder is admitted to the psychiatric unit. As the nurse approaches the client with medication, he refuses it, accusing the nurse of trying to kill him. The nurse's best strategy would be to tell him that A. "It is not poison and you must take the medication." B. "I will give you an injection if necessary." C. "You may decide if you want to take the medication by mouth or injection, but you must take it." D. "It's all right if you don't take the medication right now"

C. "You may decide if you want to take the medication by mouth or injection, but you must take it."

A nurse observes a client who has OCD repeatedly applying, removing, and reapplying makeup. The nurse identifies that repetitive behavior in a client who has OCD is due to which of the following underlying reasons? A. Adverse effect of antidepressant medication B. Fear of rejection from staff C. Attempt to reduce anxiety D. Narcissistic behavior

C. Attempt to reduce anxiety

A nurse is assisting with a court‐ordered evaluation of a client who has antisocial personality disorder. Which of the following findings should the nurse expect? (Select all that apply.) A. Demonstrates extreme anxiety when placed in a social situation B. Often engages in magical thinking C. Attempts to convince other clients to relinquish their belongings D. Becomes agitated if personal area is not neat and orderly E. Blames others for personal past and current problems

C. Attempts to convince other clients to relinquish their belongings E. Blames others for personal past and current problems

A patient admitted yesterday for injuries sustained while intoxicated believes the window blinds are snakes trying to get into the room. The patient is anxious, agitated, and diaphoretic. Which med can the RN anticipate the HCP will prescribe? A. Phenothiazine, like Thioridazine (Mellaril) B. MAOI, like Phenelzine (Nardil) C. Benzodiazepine, like Lorazepam (Ativan) D. Narcotic analgesic, like Morphine

C. Benzodiazepine, like Lorazepam (Ativan)

A patient diagnosed with bipolar disorder has rapid cycles. The health care provider prescribes an anticonvulsant medication. To prepare teaching materials, which drug should the nurse anticipate will be prescribed? A. Phenytoin (Dilantin) B. Clonidine (Catapres) C. Carbamazepine (Tegretol) D. Chlorpromazine (Thorazine)

C. Carbamazepine (Tegretol)

What is the priority nursing intervention for a patient diagnosed with delirium who has fluctuating levels of consciousness, disturbed orientation, and perceptual alterations? A. Avoidance of physical contact B. High level of sensory stimulation C. Careful observation and supervision D. Application of wrist and ankle restraints

C. Careful observation and supervision

A nurse is caring for a client who has alcohol use disorder. The client is no longer experiencing withdrawal manifestations. Which medication should the nurse anticipate administering to assist the client with maintaining abstinence from alcohol? A. Chlordiazepoxide B. Bupropion C. Disulfiram D. Carbamazepine

C. Disulfiram

The most appropriate short-term nursing goal for clients with schizophrenia is to: A. Quickly establish a warm, close relationship B. Protect client from inappropriate impulses C. Establish a trusting, nonthreatening relationship D. Set limits on bizarre behavior

C. Establish a trusting, nonthreatening relationship

A community mental health nurse is planning care to address the issue of depression among older adult clients in the community. Which of the following interventions should the nurse implement as a method of tertiary prevention? A. Educating clients on health promotion techniques to reduce the risk of depression B. Providing support groups for clients at risk for depression C. Establishing rehab programs to decrease the effects of depression D. Performing screenings for depression at a community health programs

C. Establishing rehab programs to decrease the effects of depression

A patient diagnosed with a personality disorder has used manipulation to get his or her needs met. The staff decides to apply limit setting interventions. What is the correct rationale for this action? A. It provides an outlet for feelings of anger and frustration. B. It respects the patient's wishes so assertiveness will develop. C. External controls are necessary while internal controls are developed. D. Anxiety is reduced when staff members assume responsibility for the patient's behavior.

C. External controls are necessary while internal controls are developed

34) A college student failed two tests. Afterward, the student cried and then tried to telephone a parent but got no answer. The student then gave several sweaters to a roommate. Which behavior provides the strongest clue of an impending suicide attempt? A. Calling parents B. Excessive crying C. Giving away sweaters D. Staying alone in a dorm room

C. Giving away sweaters

A client arrives for her mental health appointment wearing a cocktail dress and theatrical makeup. She announces loudly, dramatically, and in a flirtatious manner that she needs to be seen immediately because she is experiencing overwhelming psychological distress. The nurse should recognize behaviors suggestive of which personality disorder? A. Borderline B. Narcissistic C. Histrionic D. Antisocial

C. Histrionic

A client diagnosed with panic attacks is being admitted for the fifth time in 1 year because of hopelessness and helplessness. Which precaution would the nurse plan to implement? A. Elopement precautions. B. Suicide precautions. C. Homicide precautions. D. Fall precautions.

C. Homicide precautions

An adult tells the nurse, "My partner abuses me only when drinking. The drinking has increased lately, but I always get an apology afterward and a box of candy. I've considered leaving but haven't been able to bring myself to actually do it." Which phase in the cycle of violence prevents the patient from leaving? A. Tension building B. Acute battering C. Honeymoon D. Recovery

C. Honeymoon

A nurse is planning care for a client who is experiencing benzodiazepine withdrawal. Which of the following interventions should the nurse identify as the priority? A. Orient the client frequently to time, place, and person. B. Offer fluids and nourishing diet as tolerated. C. Implement seizure precautions. D. Encourage participation in group therapy sessions.

C. Implement seizure precautions

A nurse is planning a staff education program on substance use in older adults. Which of the following information should the nurse to include in the presentation? A. Older adults require higher doses of a substance to achieve a desired effect. B. Older adults commonly use rationalization to cope with a substance use disorder. C. Older adults are at an increased risk for substance use following retirement. D. Older adults develop substance use to mask manifestations of dementia.

C. Older adults are at an increased risk for substance use following retirement

A nurse should recognize that a decrease in serotonin levels would play a significant role in which mental illness? A. Bipolar disorder: mania B. Schizophrenia spectrum disorder C. Major depressive episode D. Generalized anxiety disorder

C. Major depressive episode

A patient diagnosed with Bipolar disorder is being treated on an outpatient basis with Lithium carbonate 300mg three times daily. The patient complains of nausea. To reduce the nausea, the RN can suggest that the Lithium be taken with: A. An antacid B. A large glass of juice C. Meals D. An antiemetic med

C. Meals

When used for treatment of patients diagnosed with Alzheimer disease, which medication would be expected to antagonize N-methyl-D-aspartate (NMDA) channels rather than cholinesterase? A. Donepezil (Aricept) B. Rivastigmine (Exelon) C. Memantine (Namenda) D. Galantamine (Razadyne)

C. Memantine (Namenda)

Statistically speaking, which 2 patients do you predict are at greatest risk for suicide? (SATA) A. Ms. R, a 22 yo grad student who's engaged B. Mr. M, a 34 yo male with multiple sclerosis C. Mr. A, a 68 yo Vietnam vet with TBI D. Ms. G, a 25 yo single Navajo mother who struggles with ETOH

C. Mr. A, a 68 yo Vietnam vet with TBI D. Ms. G, a 25 yo single Navajo mother who struggles with ETOH

Which is most true about elder abuse? A. Abusive caretakers are mentally ill B. Most abused older adults were abusive themselves as parents C. Often an abusive caretaker is financially dependent on the older adult in their care D. It's against the law for a caretaker to have any access to an older adult's bank account

C. Often an abusive caretaker is financially dependent on the older adult in their care

Nurse Isabelle enters the room of a client with a cognitive impairment disorder and asks what day of the week it is; what the date, month, and year are; and where the client is. The nurse is attempting to assess: A. Confabulation B. Delirium C. Orientation D. Perseveration

C. Orientation

Nurse Isabelle enters the room of a client with a cognitive impairment disorder and asks what day of the week it is; what the date, month, and year are; and where the client is. The nurse is attempting to assess: A. Confabulation B. Delirium C. Orientation D. Preservation

C. Orientation

A patient diagnosed with Schizo has catatonia. The patient has a little spontaneous movement and waxy flexibility. Which patient needs are of priority importance? A. Self-actualization B. Psychosocial C. Physiologic D. Safety and security

C. Physiologic

A person is directing traffic on a busy street while shouting and making obscene gestures at passing cars. The person has not slept or eaten for 3 days. What features of mania are evident? A. Increased muscle tension and anxiety B. Vegetative signs and poor grooming C. Poor judgment and hyperactivity D. Cognitive deficit and sad mood

C. Poor judgment and hyperactivity

A nurse is performing a mental health assessment on an adult client. According to Maslow's hierarchy of needs, which client action would demonstrate the highest achievement in terms of mental health? A. Maintaining a long-term, faithful, intimate relationship B. Achieving a sense of self-confidence C. Possessing a feeling of self-fulfillment and realizing full potential D. Developing a sense of purpose and the ability to direct activities

C. Possessing a feeling of self-fulfillment and realizing full potential

The nurse is caring for a severely depressed client who has just been admitted to the in-client psych unit. Which of the following is a PRIORITY of care? A. Nutrition B. Elimination C. Safety D. Rest

C. Safety

As a nurse prepares to administer an oral medication to a patient diagnosed with a borderline personality disorder, the patient says, "Just leave it on the table. I'll take it when I finish combing my hair." What is the nurse's best response? A. Reinforce this assertive action by the patient. Leave the medication on the table as requested. B. Respond to the patient, "I'm worried that you might not take it. I will come back later." C. Say to the patient, "I must watch you take the medication. Please take it now." D. Ask the patient, "Why don't you want to take your medication now?"

C. Say to the patient, "I must watch you take the medication. Please take it now."

A client presents in the mental health clinic saying, "I didn't expect it. They just told me this morning that I don't have a job anymore. I can't think straight. I feel like I'm going crazy." The nurse should conclude that the client is experiencing which type of crisis? A. Maturational B. Adventitious C. Situational D. Personal

C. Situational

A patient experiencing fluctuating levels of awareness, confusion, and disturbed orientation shouts, "Bugs are crawling on my legs! Get them off!" Which problem is the patient experiencing? A. Aphasia B. Dystonia C. Tactile hallucinations D. Mnemonic disturbance

C. Tactile hallucinations

A client tells a nurse, "Don't tell anyone but I hid a sharp knife under my mattress in order to protect myself from my roommate, who is always yelling at me and threatening me." Which of the following actions should the nurse take? A. Keep the client's communication confidential, but talk to the client daily, using therapeutic communication to convince him to admit to hiding the knife B. Keep the client's communication confidential, but watch the client and his roommate closely. C. Tell the client that this must be reported to the health care team because it concerns the health and safety of the client and others. D. Report the incident to the health care team, but do not inform the client of the intention to do so.

C. Tell the client that this must be reported to the health care team because it concerns the health and safety of the client and others.

A 36y/o RN who's worked in an inner city ED for 10yrs feels burnt out. He seems to be taking more sick time than normal over the past 2yrs. What might be the underlying cause of his situation? A. The RN is bored after 10yrs of being in an ED and needs to change to a different specialty B. The RN probably was exposed to a virus/bacteria and hasn't yet been diagnosed C. The RN is experiencing effects of chronic anxiety D. The RN is showing signs of PTSD

C. The RN is experiencing effects of chronic anxiety

A nurse is in the working phase of a therapeutic relationship with a client who has methamphetamine use disorder. Which of the following actions indicates transference behavior? A. The client becomes angry and threatens to engage in self-harm B. The client reminds the nurse of a friend who died from substance toxicity C. The client accuses the nurse of being controlling just like an ex-partner D. The client asks the nurse if they will go out to dinner together

C. The client accuses the nurse of being controlling just like an ex-partner

A patient diagnosed with MDD is receiving Imipramine (Tofranil) 200mg every night at bedtime. Which assessment finding would prompt the RN to collab with the HCP regarding potentially hazardous SE of this drug? A. Nasal congestion B. Dry mouth C. Urinary retention D. Blurred vision

C. Urinary retention

a nurse in an ER mental health facility is caring for a group of clients. the nurse should identify that which of the following clients requires a temporary ER admission? A. a client who has schizophrenia with delusions of grandeur B. a client who has manifestations of depression and attempted suicide a year ago C. a client who has borderline personality disorder and assaulted a homeless man with a metal rod D. a client who has bipolar disorder and paces quickly around the room while talking to himself

C. a client who has borderline personality disorder and assaulted a homeless man with a metal rod

Several children are seen in the emergency department for treatmentof illnesses and injuries. Which finding would create a high index of suspicion for child abuse? The child who has: A. repeated middle ear infections B. severe colic. C. bite marks. D. croup

C. bite marks.

The most challenging nursing intervention for patients diagnosed with personality disorders who use manipulation to get their needs met is: A. supporting behavioral change. B. monitoring suicide attempts. C. maintaining consistent limits. D. using aversive therapy.

C. maintaining consistent limits

A nurse is interviewing a client who has a new diagnosis of persistent depressive disorder. Which of the following findings should the nurse expect? A. Wide fluctuations in mood B. Report of a minimum of five clinical findings of depression C. presence of manifestations for at least 2 years D. Inflated sense of self‐esteem

C. presence of manifestations for at least 2 years

A nurse is making a home visit to a client who is in the late stage of Alzheimer's disease. The client's partner, who is the primary caregiver, wishes to discuss concerns about the client's nutrition and the stress of providing care. Which of the following actions should the nurse take? A. Verify that a current power of attorney document is on file. B. Instruct the client's partner to offer finger foods to increase oral intake. C. provide information on resources for respite care. D. Schedule the client for placement of an enteral feeding tube.

C. provide information on resources for respite care.

33) A patient experiencing acute mania undresses in the group room and dances. The nurse's first intervention would be to: A. quietly ask the patient, "Why don't you put on your clothes?" B. firmly tell the patient, "Stop dancing, and put on your clothing." C. put a blanket around the patient, and walk with the patient to a quiet room. D. allow the patient stay in the group room. Move the other patients to a different area.

C. put a blanket around the patient, and walk with the patient to a quiet room

What percentage of child molestations are perpetrated by family members? A. ~8% B. ~25% C. ~75% D. ~90%

C. ~75%

A client was recently admitted to the impatient unit after a suicide attempt and has not responded to the SSRIs or TCAs. The client asks the nurse, "I heard about MAOIs. Why can't they be added to what I'm on now? Wouldn't it help?" Which is the most appropriate nursing response? A. "There's no reason why an MAOI can't be added to your therapy" B. "ECT is your best option at this point" C. "They can't be used together because their mechanisms of action are very different" D. "Combined use can lead to a life-threatening condition called hypertensive crisis"

D. "Combined use can lead to a life-threatening condition called hypertensive crisis"

A nurse is caring for a client who is on suicide precautions. Which of the following interventions should the nurse include in the plan of care? A. Assign the client to a private room. B. Document the client's behavior every hour. C. Allow the client to keep perfume in her room. D. Ensure that the client swallows medication.

D. Ensure that the client swallows medication.

A nurse in a long‐term care facility is caring for a client who has major neurocognitive disorder and attempts to wander out of the building. The client states, "I have to get home." Which of the following statements should the nurse make? A. "You have forgotten that this is your home." B. "You cannot go outside without a staff member." C. "Why would you want to leave? Aren't you happy with your care?" D. "I am your nurse. Let's walk together to your room."

D. "I am your nurse. Let's walk together to your room."

A patient experiencing fluctuating levels of consciousness, disturbed orientation, and perceptual alteration begs, "Someone get these bugs off me." What is the nurse's best response? A. "There are no bugs on your legs. Your imagination is playing tricks on you." B. "Try to relax. The crawling sensation will go away sooner if you can relax." C. "Don't worry. I will have someone stay here and brush off the bugs for you." D. "I don't see any bugs, but I know you are frightened so I will stay with you."

D. "I don't see any bugs, but I know you are frightened so I will stay with you."

A person's spouse filed charges of battery. The person has a long history of acting-out behaviors and several arrests. Which statement by the person suggests an antisocial personality disorder? A. "I have a quick temper, but I can usually keep it under control." B. "I've done some stupid things in my life, but I've learned a lesson." C. "I'm feeling terrible about the way my behavior has hurt my family." D. "I get tired of being nagged. My spouse deserved the beating."

D. "I get tired of being nagged. My spouse deserved the beating."

A newly admitted patient diagnosed with Schizo says, "The voices are bothering me. They yell and tell me I'm bad. I have got to get away from them." Select the RN's most helpful reply: A. "Do you have a plan for getting away from the voices?" B. "Forget about the voices. Ask some other patients to sit and talk with you." C. "Do you hear the voices often?" D. "I will stay with you. Focus on what we're talking about, not the voices."

D. "I will stay with you. Focus on what we're talking about, not the voices."

A patient diagnosed with Anorexia nervosa virtually stopped eating 5 months ago and has lost 25% of body weight. A RN asks, "Describe what you think about your present weight and how you look." Which response by the patient is most consistent with the diagnosis? A. "I'm grossly underweight, but that's what I want." B. "What I think about myself is my business." C. "I'm a few lbs overweight, but I can live with it." D. "I'm fat and ugly."

D. "I'm fat and ugly."

Which individual in the emergency department should be considered at the highest risk for completing suicide? A. An adolescent Asian-American girl with superior athletic and academic skills who has asthma B. A 38-year-old single African-American female church member with fibrocystic breast disease C. A 60-year-old married Hispanic man with 12 grandchildren who has type 2 diabetes D. A 79-year-old single white man with cancer of the prostate gland

D. A 79-year-old single white man with cancer of the prostate gland

Jessie is now attending AA meetings three times a week for the past 6 weeks. She has been sober during this time and visits bi-weekly the community mental health outpatient clinic. Jessie's nurse counselor wants to talk with her about the AA meetings. Which initial question or statement is most appropriate? A. "I am so excited that you are attending the AA sessions!" B. "Do you enjoy attending the AA meetings?" C. "Are the members of the group friendly and accepting?" D. "Tell me about the last AA meeting you attended."

D. "Tell me about the last AA meeting you attended."

A patient who was referred to the eating disorders clinic has lost 35lbs in past 3 months. To assess the patient's oral intake, the RN should ask: A. "Who plans the family meals?" B. "What do you think about your present weight?" C. "Do you often feel fat?" D. "What do you eat in a typical day?"

D. "What do you eat in a typical day?"

A nurse sits with a patient diagnosed with schizophrenia. The patient starts to laugh uncontrollably, although the nurse has not said anything funny. Select the nurse's best response. A. "Why are you laughing?" B. "Please share the joke with me." C. "I don't think I said anything funny." D. "You are laughing. Tell me what's happening."

D. "You are laughing. Tell me what's happening."

When assessing a patient's plan for suicide, what aspect has priority? A. Patient's insight into suicidal motivation B. Patient's financial and educational status C. Quality and availability of patient's social support D. Availability of means and lethality of method

D. Availability of means and lethality of method

A patient committed suicide while under team care in your facility. A coworker says, "Why are we being called to a postmortem meeting? We didn't do anything wrong." What's your best explanation? A. There is almost always litigation after an inhouse suicide, & it only makes sense that someone must be held responsible. B. Staff are at high risk for hurting themselves after a suicide. C. It's important that the entire team collab to make documentation say the right things D. A postmortem assessment can help the team determine any changes that might be made in agency protocol to improve safety

D. A postmortem assessment can help the team determine any changes that might be made in agency protocol to improve safety

Which of these brain structures puts emotional meaning on a stimulus, forms emotional memories, and is involved with rage and fear? A. Temporal lobe B. Hippocampus C. Midbrain D. Amygdala

D. Amygdala

A patient diagnosed with borderline personality disorder has a history of self-mutilation and suicide attempts. The patient reveals feelings of depression and anger with life. Which type of medication should the nurse expect to administer? A. Selective serotonin reuptake inhibitor (SSRI) B. Monoamine oxidase inhibitor (MAOI) C. Benzodiazepine D. Antipsychotic

D. Antipsychotic

A patient has decreased circulating levels of GABA. Which health problem is most likely to suggest? A. Parkinson's disease B. Alzheimer's disease C. Insomnia D. Anxiety Disorders

D. Anxiety Disorders

A HCP considers which antipsychotic med to rx for a patient diagnosed with Schizo who has auditory hallucinations and poor social functioning. The patient is also overweight and has HTN. Which drug should the RN advocate? A. Ziprasidone (Geodon) B. Olanzapine (Zyprexia) C. Clozapine (Clozaril) D. Aripiprazole (Abilify)

D. Aripiprazole (Abilify)

A health care provider considers which antipsychotic medication to prescribe for a patient diagnosed with schizophrenia who has auditory hallucinations and poor social functioning. The patient is also overweight and has hypertension. Which drug should the nurse advocate? A. Clozapine (Clozaril) B. Ziprasidone (Geodon) C. Olanzapine (Zyprexa) D. Aripiprazole (Abilify)

D. Aripiprazole (Abilify)

A patient experiencing acute mania has disrobed in the hall 3 times in 2 hours. The RN should: A. Ask if the patient finds clothes bothersome B. Direct the patient to wear clothes at all times C. Tell the patient that others feel embarrassed D. Arrange for 1:1 supervision

D. Arrange for 1:1 supervision

Mrs. Chauncey, 80 years old, is taking a selective serotonin reuptake inhibitor (SSRI) and Tylenol PM daily plus other medications. She has multiple, vague somatic complaints. This morning she complains of a "stomach ache" and "gas." What is your best initial nursing response? a. Tell her to increase her water intake. b. Perform a digital rectal examination for impaction. c. Document the complaint of abdominal pain. d. Assess bowel sounds in all four quadrants.

D. Assess bowel sounds in all 4 quadrants

A patient diagnosed with schizophrenia says, "High heat. Last time here. Did you get a coat?" What type of verbalization is evident? A. Neologism B. Idea of reference C. Thought broadcasting D. Associative looseness

D. Associative looseness

What is the priority nursing intervention for a patient diagnosed with Delirium who has fluctuating LOC, disturbed orientation, and perceptual alterations? A. High level of sensory stimulation B. Avoidance of physical contact C. Application of wrist and ankle restraints D. Careful observation and supervision

D. Careful observation and supervision

Which nursing action should occur first when preparing to work with a patient who has a problem of sexual functioning? A. Acquire knowledge of the patient's sexual roles and preferences B. Develop an understanding of human sexual responses C. Assess the patient's sexual functioning D. Clarify the nurse's own personal values

D. Clarify the nurse's own personal values

Ms. S, 87, is physically disabled but otherwise healthy, except that she feels "tired. I need simplicity at this time in my life." Ms. S has no family but appoints her long-time live-in partner to act as her agent, even though she does not have a terminal illness and is not incompetent. "I'd rather have Estella make all decisions on my behalf," she says. This is most clearly an example of a(n): A. Advance directive B. Living will C. Directive to physician D. Durable power of attorney for health care

D. Durable power of attorney for health care

A 15-year-old adolescent is referred to a residential program after an arrest for theft and running away from home. At the program, the adolescent refuses to participate in scheduled activities and pushes a staff member, causing a fall. Which approach by the nursing staff would be most therapeutic? A. Neutrally permit refusals B. Coax to gain compliance C. Offer rewards in advance D. Establish firm limits

D. Establish firm limits

A medication as appropriate treatment for a client diagnosed with PTSD? A. Ziprasidone (Geodon) B. Lithium Carbonate (Lithobid) C. Lithium D. Fluoxetine (Prozac)

D. Fluoxetine (Prozac)

Which structure of the brain is involved in learning, processing information into memories, and assigning the time and place to memories? A. Olfactory bulb B. Occipital lobe C. Limbic system D. Hippocampus

D. Hippocampus

A nurse is planning care for a client who has anorexia nervosa with binge‐eating and purging behavior. Which of the following actions should the nurse include in the client's plan of care? A. Allow the client to select preferred meal times. B. Establish consequences for purging behavior. C. provide the client with a high‐fat diet at the start of treatment. D. Implement one‐to‐one observation during meal times.

D. Implement one‐to‐one observation during meal times

A nurse counseling a patient diagnosed with Dissociative Identity Disorder (DID) should understand that the assessment of highest priority is: A. Readiness to reestablish identify/ memory B. Identification of drug abuse C. Cognitive functioning D. Risk for self-harm

D. Risk for self-harm

A client takes a maintenance dosage of lithium carbonate for a bipolar disorder. She has come to the community health clinic, stating that she "has had the flu for over a week." She describes her symptoms as coughing, runny nose, chest congestion, fever, and gastrointestinal upset. Her temperature is 100.9°F. She is complaining of blurred vision and "ringing in the ears." Which situation does the nurse anticipate? A. Ingestion of foods containing tyramine B. Noncompliance with lithium therapy C. Tolerance to the lithium carbonate D. Lithium toxicity

D. Lithium toxicity

A student nurse was asked which of the following best describes dementia. Which best describes the condition? A. Memory loss occurring as part of the natural consequence of aging. B. Difficulty coping with physical and psychological change. C. Severe cognitive impairment that occurs rapidly. D. Loss of cognitive abilities, impairing ability to perform activities of daily living.

D. Loss of cognitive abilities, impairing ability to perform activities of daily living

A nurse should recognize that a decrease in norepinephrine levels would play a significant role in which mental illness? A. Bipolar disorder: mania B. Schizophrenia spectrum disorder C. Generalized anxiety disorder D. Major depressive episode

D. Major depressive episode

A nurse is caring for a client who has bipolar disorder. Which of the following is the priority nursing action? A. Set consistent limits for expected client behavior. B. Administer prescribed medications as scheduled. C. provide the client with step‐by‐step instructions during hygiene activities. D. Monitor the client for escalating behavior.

D. Monitor the client for escalating behavior

A patient diagnosed with schizophrenia has catatonia. The patient has little spontaneous movement and waxy flexibility. Which patient needs are of priority importance? A. Psychosocial B. Physiologic C. Self-actualization D. Safety and security

D. Safety and security

A patient diagnosed with AD wanders at night. Which action should the RN recommend for a family to use in the home to enhance safety? A. Obtain a bed with side rails B. Encourage daytime napping C. Place throw rugs on tile/wooden floors D. Place locks at tops of doors

D. Place locks at tops of doors

A 37-year-old is involuntarily committed to outpatient treatment after sexually molesting a 12-year-old child. The patient says, "That girl looked like she was 19 years old." Which defense mechanism is this patient using? A. Denial B. Identification C. Displacement D. Rationalization

D. Rationalization

Of the following outcomes, which one is most appropriate for a patient with cognitive impairment r/t delirium? The patient will: A. Participate fully in self-care from admission on B. Have a stable vital signs for 6hrs after admission C. Participate in simple activities that bring enjoyment D. Return to the premorbid level of functioning

D. Return to the premorbid level of functioning

Of the following outcomes, which one is most appropriate for a patient with cognitive impairment related to delirium? The patient will: A. Participate fully in self-care from admission on. B. Have stable vital signs 6 hours after admission. C. Participate in simple activities that bring enjoyment. D. Return to the premorbid level of functioning.

D. Return to the premorbid level of functioning.

A patient admitted yesterday for injuries sustained in a fall while intoxicated believes snakes are crawling on the bed. The patient is anxious, agitated, and diaphoretic. What's the priority nursing diagnosis? A .Disturbed sensory perception B. Ineffective denial C. Ineffective coping D. Risk for injury

D. Risk for injury

A newly admitted client has been diagnosed with major depressive disorder. Which nursing diagnosis takes priority? A. Social isolation R / T poor mood AEB refusing visits from family. B. Self-care deficit R / T hopelessness AEB not taking a bath for 2 weeks. C. Anxiety R / T hospitalization AEB anxiety rating of an 8/10. D. Risk for self-directed violence R / T depressed mood.

D. Risk for self-directed violence R / T depressed mood

When assessing a client for possible suicide, an important clue would be if the client A. Is hostile and sarcastic to the staff. B. Identifies with problems expressed by other C. clients. D. Seems satisfied and detached. E. Begins to talk about leaving the hospital.

D. Seems satisfied and detached

When assessing a client for possible suicide, an important clue would be if the client A. Identifies with problems expressed by other clients. B. Begins to talk about leaving the hospital. C. Is hostile and sarcastic to the staff. D. Seems satisfied and detached.

D. Seems satisfied and detached.

Which of the following will the nurse use when communicating with a client who has cognitive impairment? A. Complete explanations with multiple details. B. Pictures or gestures instead of words. C. Stimulating words and phrases to capture the client's attention. D. Short words and simple sentences.

D. Short words and simple sentences

Jessie has missed three sessions at the mental health center and sporadically attends AA meetings. She is drinking heavily this weekend and has sustained a 2-inch gash in her forehead after swerving off the road into a shallow ditch. She is taken by rescue to the ED. You are her nurse. She tells you, "I hope I just go to sleep and never wake up." What is your best intervention? A. Tell Jessie, "You are just tired and have had too much alcohol in your system." B. Clear the area of any items that Jessie may use to inflict self-harm. C. Immediately activate the mental health protocol to have Jessie admitted to a psychiatric unit. D. Stay with Jessie, call the supervisor, and arrange for continuous monitoring.

D. Stay with Jessie, call the supervisor, and arrange for continuous monitoring

The patient's family reports that the patient has become irritable, isn't sleeping, and tells them he can't relax. This has been ongoing for 2 days. Family says he hasn't left the house in 2 weeks. Family member tells you there are empty beer cans all over the place. He assures you that he looked everywhere and there's no alcohol in the house, just the empty cans. What problem does the patient exhibit? A. ETOH abuse B. Agoraphobia C. Panic attack D. Substance abuse anxiety disorder

D. Substance abuse anxiety disorder

A RN uses the modified SAD PERSONS scale to interview a patient. This tool provides data relevant to: A. Level of anxiety B. Current stress level C. Mood disturbance D. Suicide potential

D. Suicide potential

A nurse is assessing a client diagnosed with pedophiliac disorder. What would differentiate this sexual disorder from a sexual dysfunction? A. Symptoms of sexual dysfunction include inappropriate sexual behaviors, whereas symptoms of a sexual disorder include impairment in normal sexual response B. Sexual disorders can be caused by decreased levels of circulating androgens, whereas levels of circulating androgens don't affect sexual dysfunction C. Sexual dysfunction can be caused by increasing levels of circulating androgens, whereas levels of circulating androgens don't affect sexual disorders D. Symptoms of sexual disorder include inappropriate sexual behaviors, whereas symptoms of a sexual dysfunction include impairment in normal sexual response

D. Symptoms of sexual disorder include inappropriate sexual behaviors, whereas symptoms of a sexual dysfunction include impairment in normal sexual response

A 5-year-old child diagnosed with attention deficit hyperactivity disorder (ADHD) bounces out of a chair in the waiting room, runs across the room, and begins to slap another child. What is the nurse's best action? A. Call for emergency assistance from another staff member. B. Instruct the parents to take the child home immediately. C. Direct this child to stop, and then comfort the other child. D. Take the child into another room with toys to act out feelings.

D. Take the child into another room with toys to act out feelings.

A 65y/o woman reports periods of palpitations, sweating, and slight SOB. She feels like she's going to die. You speak with her daughter who reports that her father recently passed away and her mother lives alone. How would you respond? A. Tell the daughter that her mother is likely experiencing depressive disorder B. Tell the daughter that her mother is likely experiencing panic attacks C. Tell the daughter that her mother is likely experiencing panic attacks and will develop depressive disorder D. Tell the daughter that her mother is likely experiencing panic attacks and that she should tell her PCP about it so it can be treated soon

D. Tell the daughter that her mother is likely experiencing panic attacks and that she should tell her PCP about it so it can be treated soon

A patient diagnosed with major depressive disorder is receiving imipramine (Tofranil) 200 mg every night at bedtime. Which assessment finding would prompt the nurse to collaborate with the health care provider regarding potentially hazardous side effects of this drug? A. Dry mouth B. Blurred vision C. Nasal congestion D. Urinary retention

D. Urinary retention

When preparing to interview a patient diagnosed with narcissistic personality disorder, a nurse can anticipate the assessment findings will include: A. preoccupation with minute details; perfectionism. B. charm, drama, seductiveness; seeking admiration. C. difficulty being alone; indecisiveness, submissiveness. D. grandiosity, attention seeking, and arrogance.

D. grandiosity, attention seeking, and arrogance

After the death of a spouse, an adult repeatedly says, "I should have made him go to the doctor when he said he didn't feel well." This individual is experiencing: A. preoccupation with the image of the deceased. B. sensations of somatic distress. C. anger. D. guilt.

D. guilt

67) A nurse is caring for a client following the loss of a partner due to a terminal illness. Identify the sequence of Engel's five stages of grief that the nurse should expect the client to experience. (Select the stages of grief in order of occurrence.) A. Restitution B. Recovery C. Shock and disbelief D. Resolution of the loss E. Developing awareness

Shock and disbelief Developing awareness Restitution Resolution of the loss Recovery

A patient asks for information about Alcoholics Anonymous (AA). Which is the nurse's best response? a. "It is a self-help group with the goal of sobriety." b. "It is a form of group therapy led by a psychiatrist." c. "It is a group that learns about drinking from a group leader." d. "It is a network that advocates strong punishment for drunk drivers."

a. "It is a self-help group with the goal of sobriety."

When a patient diagnosed with schizophrenia was discharged 6 months ago, haloperidol (Haldol) was prescribed. The patient now says, "I stopped taking those pills. They made me feel like a robot." What are common side effects the nurse should validate with the patient? a. Sedation and muscle stiffness b. Sweating, nausea, and diarrhea c. Mild fever, sore throat, and skin rash d. Headache, watery eyes, and runny nose

a. Sedation and muscle stiffness

When a person first begins drinking alcohol, two drinks produce relaxation and drowsiness. After one year of drinking, four drinks are needed to achieve the same relaxed, drowsy state. Why does this change occur? a. Tolerance develops. b. The alcohol is less potent. c. Antagonistic effects occur. d. Hypomagnesemia develops.

a. Tolerance develops.

Mrs. H, 87, is anxious. She tells you she must go home immediately, saying: "My twins need me. They're barely a year old!" Select the best response. a. Help reorient her by explaining patiently that she is too old now to still have babies. b. Ask her questions to describe her need to go home and sympathize with how hard it can be to be away from home. c. Implement withdrawal and promise to return in 10 minutes when she is calmer and more rational. d. Reward her with attention when she focuses on reality.

b. Ask her questions to describe her need to go home and sympathize with how hard it can be to be away from home.

A patient's care plan includes monitoring for auditory hallucinations. Which assessment findings suggest the patient may be hallucinating? a. Elevated moos, hyperactivity, distractibility b. Darting eyes, tilted head, mumbling to self c. Aloofness, haughtiness, suspicion d. Performing rituals, avoiding open places

b. Darting eyes, tilted head, mumbling to self

Consider these health problems: Lewy body disease, Pick disease, and Korsakoff syndrome. Which term unifies these problems? a. Intoxication b. Dementia c. Delirium d. Amnesia

b. Dementia

Which change in the brain's biochemical function is most associated with suicidal behavior? a. Dopamine excess b. Serotonin deficiency c. Acetylcholine excess d. Gamma-aminobutyric acid deficiency

b. Serotonin deficiency

Mrs. Chauncey, 80 years of age, complains of stomach pain and is now mute and staring out of her window. She is refusing food. Which of the following interventions are appropriate? (Select all that apply.) a. Give her privacy, and close her door. b. Speak with her, although she may not answer. c. Continue to offer her food and fluids. d. Regularly assess vital signs and skin turgor.

b. Speak with her, although she may not answer c. Continue to offer her food and fluids d. Regularly assess vital signs and skin turgor

What behavior by a nurse caring for a patient diagnosed with an eating disorder indicates the nurse needs supervision? a. The nurses comments are nonjudgmental. b. The nurse uses an authoritarian manner when interacting with the patient. c. The nurse teaches the patient to recognize signs of increasing anxiety and ways to intervene. d. The nurse refers the patient to a self-help group for individuals with eating disorders.

b. The nurse uses an authoritarian manner when interacting with the patient.

When you hear the 3-11 shift report, you learn that one of your patients was aggressive during a manic phase and restrained (wrists and ankles) in the seclusion room. Which nursing action is your top priority? a. Offer fluids, a snack, and toileting. b. Wake your patient, and assess vital signs. c. Check each extremity for circulation. d. Check the electronic medication administration record (e-MAR) for recently administered scheduled and as-needed medications. e. Assess mental status.

b. Wake your patient, and assess vital signs

Conrad, a 26-year-old male, is being treated for cuts after being arrested for breaking the windows in the home of his girlfriend, who has just told him that they are through. He has a history of having been physically abused by his father, setting fires as a child, treatment for substance abuse, and arrest for stealing a car. Which of the following personality disorders might the clinician consider? a) Narcissistic b) Paranoid c) Antisocial d) Histrionic

c) Antisocial

Over the past year, a woman has cooked gourmet meals for her family but eats only tiny servings. This person wears layered loose clothing. Her current weight is 95 pounds, a loss of 35 pounds. Which medical diagnosis is most likely? a. Binge eating b. Bulimia nervosa c. Anorexia nervosa d. Pica

c. Anorexia nervosa

As a patient admitted to the eating disorders unit undresses, a nurse observes that the patient's body is covered by fine, downy hair. The patient weighs 70 pounds and is 5 feet 4 inches tall. Which term should be documented? a. Amenorrhea b. Alopecia c. Lanugo d. Stupor

c. Lanugo

Mrs. Chauncey receives a visit from her priest. He runs out of her room and then pulls the nurse assistant back into her room. Mrs. Chauncey is cutting her left wrist (superficially) with the 5 x 7 glass from a framed photo of a grandchild. She is taken to the emergency department, where her wrist is bandaged. Her daughter and son-in-law are notified. As her nurse, which of the following statements help clarify what has taken place? a. "Don't worry, I think your mom is just confused." b. "Your mom has been more withdrawn over the last few days." c. "I am very concerned that your mom is suicidal." d. "When your mom's priest arrived, he found her cutting her wrist with the glass from a framed photo."

d. "When your mom's priest arrived, he found her cutting her wrist with the glass from a framed photo."

Mr. Quang is dancing under the overhead television of the crisis stabilization unit and taunting the other patients in the room. He shouts, "I own the TV networks, so they have to do what I say!" As Mr. Quang's nurse, what is your best initial intervention at this time? a. Leave him alone, and remove the other patients. b. Tell Mr. Quang that he has to obey the rules, or he will be restrained. c. Medicate Mr. Quang with an anxiolytic agent, and place him in seclusion. d. Calmly motion for Mr. Quang to come with you to the dining room for a snack.

d. Calmly motion for Mr. Quang to come with you to the dining room for a snack


Conjuntos de estudio relacionados

تمرينات 3ث رياضيات - محمود نصار

View Set

Chapter 12 wrist/hand & injuries

View Set

Anatomy & Physiology 1 McGrawHill Homework Chapter 11

View Set